Last visit was: 27 Apr 2024, 07:43 It is currently 27 Apr 2024, 07:43

Close
GMAT Club Daily Prep
Thank you for using the timer - this advanced tool can estimate your performance and suggest more practice questions. We have subscribed you to Daily Prep Questions via email.

Customized
for You

we will pick new questions that match your level based on your Timer History

Track
Your Progress

every week, we’ll send you an estimated GMAT score based on your performance

Practice
Pays

we will pick new questions that match your level based on your Timer History
Not interested in getting valuable practice questions and articles delivered to your email? No problem, unsubscribe here.
Close
Request Expert Reply
Confirm Cancel
SORT BY:
Kudos
Math Expert
Joined: 02 Sep 2009
Posts: 92952
Own Kudos [?]: 619394 [14]
Given Kudos: 81611
Send PM
VP
VP
Joined: 20 Jul 2017
Posts: 1300
Own Kudos [?]: 3455 [4]
Given Kudos: 162
Location: India
Concentration: Entrepreneurship, Marketing
GMAT 1: 690 Q51 V30
WE:Education (Education)
Send PM
GMAT Club Legend
GMAT Club Legend
Joined: 08 Jul 2010
Status:GMAT/GRE Tutor l Admission Consultant l On-Demand Course creator
Posts: 5962
Own Kudos [?]: 13394 [1]
Given Kudos: 124
Location: India
GMAT: QUANT+DI EXPERT
Schools: IIM (A) ISB '24
GMAT 1: 750 Q51 V41
WE:Education (Education)
Send PM
Target Test Prep Representative
Joined: 14 Oct 2015
Status:Founder & CEO
Affiliations: Target Test Prep
Posts: 18768
Own Kudos [?]: 22070 [0]
Given Kudos: 283
Location: United States (CA)
Send PM
Re: List Q consists of {12, 17, 21, 24, 30 and x}. For how many integer va [#permalink]
Expert Reply
Bunuel wrote:
List Q consists of {12, 17, 21, 24, 30 and x}. For how many integer values of x is the mean of S equal to the median of S?

(A) None
(B) One
(C) Two
(D) Three
(E) More than three



Solution:

The mean is (12 + 17 + 21 + 24 + 30 + x)/6 = (104 + x)/6. Since there are 6 values, the median is the average of the two values in the middle. It could be one of the following:

(17 + 21)/2 = 19 (if x is the smallest value or the second smallest value)

(x + 21)/2 (if x is the third smallest value or the third largest value)

(21 + 24)/2 = 22.5 (if x is the second largest value or the largest value)

We are given that the mean is equal to the median. If the median is 19, we have:

(104 + x)/6 = 19

104 + x = 114

x = 10

We see that if x = 10, it will be indeed the smallest value in the set. So 10 could be a value of x.

If the median is (x + 21)/2, we have:

(104 + x)/6 = (x + 21)/2

6(x + 21) = 2(104 + x)

6x + 126 = 208 + 2x

4x = 82

x = 20.5

Since 20.5 is not an integer, 20.5 could not be a value of x.

If the median is 22.5, we have:

(104 + x)/6 = 22.5

104 + x = 135

x = 31

We see that if x = 31, it will be indeed the largest value in the set. So 31 could be a value of x.

Answer: C
User avatar
Non-Human User
Joined: 09 Sep 2013
Posts: 32717
Own Kudos [?]: 822 [0]
Given Kudos: 0
Send PM
Re: List Q consists of {12, 17, 21, 24, 30 and x}. For how many integer va [#permalink]
Hello from the GMAT Club BumpBot!

Thanks to another GMAT Club member, I have just discovered this valuable topic, yet it had no discussion for over a year. I am now bumping it up - doing my job. I think you may find it valuable (esp those replies with Kudos).

Want to see all other topics I dig out? Follow me (click follow button on profile). You will receive a summary of all topics I bump in your profile area as well as via email.
GMAT Club Bot
Re: List Q consists of {12, 17, 21, 24, 30 and x}. For how many integer va [#permalink]
Moderators:
Math Expert
92952 posts
Senior Moderator - Masters Forum
3137 posts

Powered by phpBB © phpBB Group | Emoji artwork provided by EmojiOne